The base of a triangle is 3 inches shorter than its height. Its area is 275 square inches. Set up a quadratic equation and solve to find its base and height.

Answers

Answer 1

Answer: hope its help

Let's start by assigning variables to the unknown quantities in the problem. Let h be the height of the triangle in inches, and let b be the base of the triangle in inches.

According to the problem, the base of the triangle is 3 inches shorter than its height. This can be expressed as:

b = h - 3

The formula for the area of a triangle is:

A = (1/2)bh

We are given that the area of the triangle is 275 square inches, so we can substitute these values into the formula to get:

275 = (1/2)(h)(h-3)

Simplifying the right-hand side, we get:

275 = (1/2)(h^2 - 3h)

Multiplying both sides by 2 to eliminate the fraction, we get:

550 = h^2 - 3h

Rearranging this equation to standard quadratic form, we get:

h^2 - 3h - 550 = 0

Now we can solve for h using the quadratic formula:

h = (-b ± sqrt(b^2 - 4ac)) / (2a)

In this case, a = 1, b = -3, and c = -550, so we can substitute these values into the formula to get:

h = (-(-3) ± sqrt((-3)^2 - 4(1)(-550))) / (2(1))

Simplifying the expression inside the square root, we get:

h = (3 ± sqrt(2209)) / 2

We can ignore the negative solution since height must be positive, so we get:

h = (3 + sqrt(2209)) / 2 ≈ 29.04

Now that we know the height of the triangle is approximately 29.04 inches, we can use the equation b = h - 3 to find the length of the base:

b = 29.04 - 3 = 26.04

Therefore, the base of the triangle is approximately 26.04 inches, and the height is approximately 29.04 inches.

Step-by-step explanation:


Related Questions

he buys a 5kg lwisa samp and repacks the samp into 125g packets. determine how many packets will be able to get from one pack of 5kg samp?​

Answers

Answer:5

Step-by-step explanation:5555555

HELP ME ASAPPP!!! Ill mark you brasinlisest

Answers

Step-by-step explanation:

Theoretical probabilities can be calculated using the concept of probability. Each student has a 0.5 chance of selecting either List A or List B. Therefore, the probability of getting 27 heads and 33 tails can be calculated as:

P(27 heads and 33 tails) = (60 choose 27) * (0.5)^27 * (0.5)^33 where (60 choose 27) is the number of ways to select 27 students out of 60.

Using a calculator, we can compute the above probability as approximately 0.109. This means that if we were to repeat this experiment many times, we would expect to get 27 heads and 33 tails about 10.9% of the time.

Comparing this theoretical probability to the experimental results, we see that the observed proportion of heads (27/60 = 0.45) is lower than the expected proportion of heads (0.5) and the observed proportion of tails (33/60 = 0.55) is higher than the expected proportion of tails (0.5).

However, it is important to note that due to the random nature of the experiment, we would not expect the exact theoretical probabilities to match the experimental results exactly. In other words, there is always some amount of variation expected in the results. Nonetheless, the experimental results are consistent with the theoretical probabilities, and we can conclude that there is no significant deviation from what we would expect by chance.

Help pleaseeee!!


On January 1, 2014, the federal minimum wage was $7.25 per hour. Which graph has a slope that best represents this rate?

Answers

The horizontal line at $7.25 on the y-axis of the graph is the one with a slope that most accurately depicts the federal minimum wage of $7.25 per hour as of January 1, 2014.

Which federal minimum wage was the highest?

Although it varies from state to state, the federally mandated minimum wage in the United States is $7.25 per hour. The District of Columbia had the highest minimum wage in the US as of January 1, 2023, at 16.50 dollars per hour.

How are minimum wages determined?

The variable dearness allowance (VDA) component, which takes into account inflationary trends, such as an increase or fall in the Consumer Price Index (CPI), and, if applicable, the housing rent, are included in the computation of the monthly minimum salary.

To know more about graph visit:-

https://brainly.com/question/17267403

#SPJ1

Below is the graph of a trigonometric function. It has a minimum point at

(1, 1.5) and an amplitude of 1.5. What is the midline equation of the function?

Answers

The midline equation of the function is y = 1.5.

What is amplitude of trigonometric functions?

The gap between a trigonometric function's highest and least values is known as its amplitude. The difference between the greatest and minimum numbers is, in other words, divided by two. For instance, the amplitude is A in the equation y = A sin(Bx) + C. The amplitude, also known as the average value of the function across a period, denotes the "height" of the function above and below the midline. It gauges the magnitude or intensity of the oscillation of the function's representation of. The oscillation is more prominent and subtler depending on the amplitude, which ranges from higher to lower values.

Given that, the function has minimum point at (1, 1.5) and an amplitude of 1.5.

Using the definition of the amplitude the midline is the given amplitude.

Hence, the midline equation of the function is y = 1.5.

Learn more about amplitude here:

https://brainly.com/question/9124856

#SPJ1

Answer:

3

Step-by-step explanation:

Khan Academy

Martin Pincher purchased a snow shovel for $28.61, a winter coat for $23.27, and some rock salt for $7.96. He must pay the state tax of 5 percent, the county tax of 0.5 percent and the city tax of 2.5 percent. What is the total purchase price?

Answers

Hi Martin,

To calculate the total purchase price of your items, you'll need to apply the state, county, and city taxes to the total purchase cost of all three items.

The total purchase cost of all three items is:

Snow shovel: $28.61

Winter coat: $23.27

Rock salt: $7.96

Total purchase cost: $59.84

Now, we apply the applicable taxes:

State tax: 5% of $59.84 = $2.99

County tax: 0.5% of $59.84 = $0.30

City tax: 2.5% of $59.84 = $1.49

Total taxes: $2.99 + $0.30 + $1.49 = $4.78

Therefore, the total purchase price is:

Total purchase cost + Total taxes = $59.84 + $4.78 = $64.62

What rotation centered about the origin maps (4, − 7) to (7,4) ? 90° counterclockwise 180° counterclockwise 270° counterclockwise I don't know. ←​

Answers

Answer:

What rotation centered about the origin maps (4, − 7) to (7,4) ? 90° counterclockwise 180° counterclockwise 270° counterclockwise I don't know. ←​

Step-by-step explanation:

To map the point (4, -7) to (7, 4) by a rotation centered about the origin, we need to find the angle of rotation and direction.

We can start by finding the vector from the origin to (4, -7), which is <4, -7>. We want to rotate this vector to the vector from the origin to (7, 4), which is <7, 4>.

To do this, we need to find the angle between these two vectors. Using the dot product, we have:

<4, -7> · <7, 4> = (4)(7) + (-7)(4) = 0

Since the dot product is zero, we know that the two vectors are orthogonal, and the angle between them is 90 degrees.

To map (4, -7) to (7, 4) with a 90-degree rotation counterclockwise, we can use the matrix:

[0 -1]

[1 0]

Multiplying this matrix by the vector <4, -7>, we get:

[0 -1] [4] = [-7]

[1 0] [-7] [ 4]

which corresponds to the point (-7, 4). This matches our desired endpoint, so the answer is 90° counterclockwise.

Answer:

90° counterclockwise

Step-by-step explanation:

I am not sure if the picture helps or not.  I am trying to show that I traced the point (4,-7).  Then I have a plus sign at (0,0).  I start rotating the tracing paper counterclockwise until  I get to the point (7,4).  I needed to turn one turn of the plus sign.  That would be 90°

Helping in the name of Jesus.

Twelve of the workers received the following salaries: three of them earn P12,500 a month, four of them earn P11,000; twoearn P10,500 and the rest earn P9,000 a month. What is the median salary of the workers?

Answers

The median salary of the workers is P11,000.

What is median?

In statistics, the median is a measure of central tendency that represents the middle value of a data set when the data set is ordered from least to greatest (or vice versa). If the data set has an odd number of values, the median is the middle value. If the data set has an even number of values, the median is the average of the two middle values. The median is used as a measure of central tendency when the data set has outliers or is not normally distributed.

How to calculate median?

To calculate the median of a set of numbers:

Put the numbers in order from lowest to highest.If the number of items in the list is odd, the median is the middle number. For example, if the list is 3, 5, 7, 9, 11, the median is 7.If the number of items in the list is even, the median is the average of the two middle numbers. For example, if the list is 4, 6, 8, 10, the median is (6 + 8)/2 = 7.

In the given question,

To find the median salary of the workers, we need to arrange the salaries in order from lowest to highest.

The salaries are:

P9,000, P9,000, P10,500, P10,500, P11,000, P11,000, P11,000, P11,000, P12,500, P12,500, P12,500, P12,500

There are 12 workers, so the median salary will be the average of the 6th and 7th salaries when arranged in order.

Median salary = (P11,000 + P11,000)/2

= P11,000

Therefore, the median salary of the workers is P11,000.

To know more median, visit:

https://brainly.com/question/28060453

#SPJ1

The odometer in Mr. Washington's car does not work correctly. The odometer recorded 14.3 miles for his last trip to the hardware store,but he knows the distant traveled is 17 miles.What is the percent error. Show steps


Find the percent errors, use the formula a-x/x ×100%

Answers

Answer:

15.8823% error

Step-by-step explanation:

Percent Error Formula

abs([tex]\frac{(a-x)}{x}[/tex]) * 100%

abs:  absolute value

a:  actual value

x:  expected value

Percent Error = abs([tex]\frac{14.3 - 17}{17}[/tex]) * 100%

                       = abs(-0.158823) * 100%

                       15.8823% error

can someone help me? please
evalute the following function h(x)=3x2+ax-1 for h(3) and find the value for a.​

Answers

Answer:

Step-by-step explanation:

[tex]h(3)=3\times 3^2+3a-1 \rightarrow h(3)=26+3a[/tex]

But we cannot find [tex]a[/tex] unless we are told what [tex]h(3)[/tex] equals.

Help I don’t just this

Answers

The conditional relative frequency that a student rides the bus, given that the student is in middle school is 0.16 / 0.96 ≈ 0.17.

Describe conditional relative frequency ?

Conditional relative frequency is a statistical measure that describes the proportion or percentage of a specific group or category within a subset of data. It is calculated by dividing the frequency of the specific category in the subset by the frequency of the total subset.

To find the conditional relative frequency that a student rides the bus, given that the student is in middle school, we need to divide the frequency of middle school students who ride the bus by the total number of middle school students.

From the table, we see that the frequency of middle school students who ride the bus is 0.16. The total number of middle school students is the sum of the frequencies in the first row, which is 0.20 + 0.16 + 0.12 + 0.48 = 0.96.

So, the conditional relative frequency that a student rides the bus, given that the student is in middle school is:

0.16 / 0.96 ≈ 0.17

Rounded to the nearest hundredth, the answer is 0.17. Therefore, about 17% of middle school students ride the bus.

To know more about frequency visit:

https://brainly.com/question/29269840

#SPJ1

The question is in the picture.

Answers

The composite function is given as follows: N(T(t)) = 720t² - 480t - 175.8.The time needed for the battery to reach 18413 bacteria is given as follows: 4.76 hours.

How to obtain the composite function?

The composite function is obtained applying the inner function as the input to the outer function.

The functions for this problem are defined as follows:

Inner function: T(t) = 6t + 1.2.Outer function: N(T) = 20T² - 128t + 77.

Hence the composite function is obtained replacing the two instances of T on the function N(t) by the definition of T(t), hence:

N(T(t)) = 20(6t + 1.2)² - 128(6t + 1.2) + 77

N(T(t)) =  720t² + 288t + 28.8 - 768t - 204.6.

N(T(t)) = 720t² - 480t - 175.8.

For a population of 18413 bacteria, we have that N(T(t)) = 18413, hence:

720t² - 480t - 175.8 = 18413

720t² - 480t - 18588.8 = 0.

The coefficients of the quadratic function are given as follows:

a = 720, b = 480, c = -18.588.8.

Using a quadratic function calculator, the positive solution is given as follows:

t = 4.76 hours.

More can be learned about composite functions at https://brainly.com/question/10687170

#SPJ1

locate the absolute extrema of the function
on the closed interval

Answers

Answer:

To find the integral of f(x) = 2x + 5/3 over the interval [0, 5], we can use the definite integral formula:

∫[a,b] f(x) dx = F(b) - F(a)

where F(x) is the antiderivative of f(x).

First, we find the antiderivative of f(x):

F(x) = x^2 + (5/3)x + C

where C is the constant of integration.

Next, we evaluate F(5) and F(0):

F(5) = 5^2 + (5/3)(5) + C = 25 + (25/3) + C

F(0) = 0^2 + (5/3)(0) + C = 0 + 0 + C

Subtracting F(0) from F(5), we get:

∫[0,5] f(x) dx = F(5) - F(0)

= 25 + (25/3) + C - C

= 25 + (25/3)

= 100/3

Therefore, the definite integral of f(x) = 2x + 5/3 over the interval [0, 5] is 100/3.

Please answer the attached question

Answers

The values of e and f in the given equation are: e = 2√3 ± √(4√3), e = 2√3 ± 2√2, and f = 4√3.

How are radicals solved?

Equations containing radicals can be made simpler by solving the resultant equation after squaring both sides of the equation to remove the radical. Nonetheless, caution must be exercised to guarantee that any solutions found are reliable and adhere to any variables' limitations.

The given equation is [tex](e - 2\sqrt{3} )^2[/tex] = f - 20√3.

Expanding the left side of the equation we have:

[tex](e - 2\sqrt{3} )^2[/tex] = (e - 2√3)(e - 2√3)

= [tex]e^2[/tex] - 2e√3 - 2e√3 + 12

= [tex]e^2[/tex] - 4e√3 + 12

Substituting back in the function

[tex]e^2[/tex] - 4e√3 + 12 = f - 20√3

[tex]e^2[/tex] - 4e√3 - f + 20√3 - 12 = 0

Using the quadratic formula:

e = [4√3 ± √(16*3 + 4(f - 20√3 + 12))] / 2

e = [4√3 ± √(4f - 64√3)] / 2

e = 2√3 ± √(f - 16√3)

Now for,

(e - 2√3)² = f - 20√3

(2√3 + √(f - 16√3) - 2√3)² = f - 20√3

f - 20√3 = f - 16√3

f = 4√3

Hence, the values of e and f in the given equation are: e = 2√3 ± √(4√3), e = 2√3 ± 2√2, and f = 4√3.

Learn more about radicals here:

brainly.com/question/1369233

#SPJ1

Which of the following is true regarding cross-sectional data sets? Check all that apply. The data consist of a sample of multiple individuals. It can be assumed that the data were obtained through a random sampling of the underlying population. These data require attention to the frequency at which they are collected (weekly, monthly, yearly, etc.). The data are collected at approximately the same point in time.

Answers

The correct options are:

The data contain a sample of multiple individuals.

The data are collected  are approximately at  the same point in time.

Cross-sectional data sets are a type of research design used in statistical analysis. They consist of a sample of multiple individuals or entities observed at a single point in time. One of the characteristics of cross-sectional data sets is that they do not involve any observation or measurement over time, making them different from longitudinal or time-series data sets.

One advantage of cross-sectional data sets is that they are relatively easy and inexpensive to collect. It can be assumed that the data were obtained through a random sampling of the underlying population, making them representative of the larger population. However, these data require attention to the frequency at which they are collected (weekly, monthly, yearly, etc.), as the timing of data collection can impact the results.

Overall, cross-sectional data sets can provide a snapshot of a population or phenomenon at a specific point in time, making them useful for a wide range of research questions in social, economic, and political sciences.

Find out more about Cross-sectional data

at brainly.com/question/14364178

#SPJ4

f(x)
g(x)
=6x−4
=3x
2
−2x−10
Escribe (g∘f)(x) como una expresión en términos de x.

Answers

Answer:

g(x)

=6x−4

=3x

2

−2x−10

Escribe (g∘f)(x) como una expresión en términos de x.

Step-by-step explanation:

Primero necesitamos conocer la función f(x). Luego podemos sustituir f(x) en g(x) para obtener (g∘f)(x).

Como la función f(x) no se proporcionó en la pregunta, asumiré que f(x) es:

f(x) = x^2 - 2x + 1

Entonces, podemos sustituir f(x) en g(x) de la siguiente manera:

g(f(x)) = 6f(x) - 4

= 6(x^2 - 2x + 1) - 4 (sustituyendo f(x))

= 6x^2 - 12x + 2

Por lo tanto, (g∘f)(x) = 6x^2 - 12x + 2.

How many proper subsets are in {2,4,6,8...100}

Answers

Answer:

159 proper subsets.

Step-by-step explanation:

Given a set {2, 4, 6, 8...100}, how many proper subsets are there?

First, find how many subsets there are in 2 - 10:

That's 16.

Then because there are 10 10s in 100, multiply by 10:

16 x 10 = 160

Finally, because it says proper subsets, subtract by 1:

160 - 1 = 159 proper subsets.

Therefore, there are 159 proper subsets in {2, 4, 6, 8...100}

A bookcase contains 2 statistics books and 5 biology books. If 2 books are chosen at random, the chance that both are statistics books isA 1 / 21B 10 / 21C 11D 21 / 11

Answers

If 2 books are chosen at random, then the probability that both are statistics books is (a) 1/21.

The number of statistics book in bookcase is = 2;

The number of biology books in bookcase is = 5;

So, the total number of books is = 7;

The Probability of choosing a statistics book on the first draw is 2/7, since there are 2 statistics books out of a total of 7 books.

After the first book is chosen, there will be 6 books left, including 1 statistics book out of a total of 6 books.

So, the probability of choosing another statistics book on the second draw is 1/6.

In order to find the probability of both events happening together (i.e. choosing 2 statistics books in a row), we multiply the probabilities of each event:

So, P(choosing 2 statistics books) = P(1st book is statistics) × P(2nd book is statistics given that the 1st book was statistics);

⇒ (2/7) × (1/6)

⇒ 1/21

Therefore, the required probability is (a) 1/21.

Learn more about Probability here

https://brainly.com/question/13683807

#SPJ4

The given question is incomplete, the complete question is

A bookcase contains 2 statistics books and 5 biology books. If 2 books are chosen at random, the chance that both are statistics books is

(a) 1/21

(b) 10/21

(c) 11

(d) 21/11

How do you write 0.38 as a percentage?

Write your answer using a percent sign (%).

Answers

Answer:

38%

Step-by-step explanation:

I learned in class on Friday.

Multiply both numerator and denominator by 100. We do this to find an equivalent fraction having 100 as the denominator.

[tex]0.38\times \dfrac{100}{100}[/tex]

[tex]= (0.38 \times 100) \times \dfrac{1}{100} =\dfrac{38}{100}[/tex]

Write in percentage notation: 38%

A simple random sample with n = 25 provided a sample mean of 30 and a sample standard deviation of 4. Assume the population is approximately normal. a. Develop a 90% confidence interval for the population mean. b. Develop a 95% confidence interval for the population mean. c. Develop a 99% confidence interval for the population mean. d. What happens to the margin of error and the confidence interval as the confidence level is increased?

Answers

Conversely, as the confidence level decreases, the margin of error becomes smaller, and the confidence interval becomes narrower.

What is confidence interval?

In statistics, a confidence interval is a range of values that is likely to contain the true value of a population parameter (such as a mean or a proportion), based on a sample from that population. The confidence interval is typically expressed as an interval around a sample statistic, such as a mean or a proportion, and is calculated using a specified level of confidence, typically 90%, 95%, or 99%.

Here,

To develop a confidence interval, we need to use the following formula:

Confidence Interval = sample mean ± margin of error

where the margin of error is calculated as:

Margin of Error = z* (sample standard deviation/ √n)

where z* is the critical value from the standard normal distribution table based on the chosen confidence level.

a. For a 90% confidence interval, the critical value (z*) is 1.645. Thus, the margin of error is:

Margin of Error = 1.645 * (4 / √25) = 1.317

So, the 90% confidence interval for the population mean is:

30 ± 1.317, or (28.683, 31.317)

b. For a 95% confidence interval, the critical value (z*) is 1.96. Thus, the margin of error is:

Margin of Error = 1.96 * (4 / √25) = 1.568

So, the 95% confidence interval for the population mean is:

30 ± 1.568, or (28.432, 31.568)

c. For a 99% confidence interval, the critical value (z*) is 2.576. Thus, the margin of error is:

Margin of Error = 2.576 * (4 / √25) = 2.0656

So, the 99% confidence interval for the population mean is:

30 ± 2.0656, or (27.9344, 32.0656)

d. As the confidence level increases, the margin of error also increases, because we need to be more certain that our interval includes the true population mean. This means that the confidence interval becomes wider as the confidence level increases.

To know more about confidence interval,

https://brainly.com/question/28969535

#SPJ1

Census data for a certain county shows that 19% of the adult residents are Hispanic. Suppose 92 people are called to jury duty and only 11 of them are Hispanic. Does this apparent underrepresentation of Hispanics call into question the fairness of the jury selection process? Again run a test using the PHANTOMS method to complete all parts of your problem.

Answers

Yes, the apparent under representation of Hispanics on the jury duty calls into question the fairness of the jury selection system, as the results of the hypothesis test suggest that the system may be systematically excluding Hispanics.

To determine whether the apparent underrepresentation of Hispanics on the jury selection system is statistically significant and calls into question its fairness, we need to perform a hypothesis test.

First, we set up the null hypothesis, which is the assumption that there is no difference between the proportion of Hispanics in the county population and the proportion of Hispanics in the jury pool. That is, the proportion of Hispanics in the jury pool is expected to be equal to 19%.

The alternative hypothesis is that there is a difference between the proportion of Hispanics in the county population and the proportion of Hispanics in the jury pool.

We can use a one-tailed z-test to test the null hypothesis, where the test statistic is calculated as:

z = (p - P) / sqrt(P(1-P)/n)

where p is the proportion of Hispanics in the jury pool, P is the proportion of Hispanics in the county population (0.19), and n is the sample size (72).

Plugging in the values, we get

z = (9/72 - 0.19) / sqrt(0.19*(1-0.19)/72) = -2.39

Assuming a significance level of 0.05, we compare the calculated z-value with the critical z-value of -1.645 (obtained from a standard normal distribution table). Since the calculated z-value is less than the critical z-value, we reject the null hypothesis and conclude that the proportion of Hispanics in the jury pool is significantly different from the proportion of Hispanics in the county population.

Learn more about hypothesis test here

brainly.com/question/30588452

#SPJ4

The graph of y = 5x2 is

Answers

Answer:

................................

What does the point (5, 10) represent on the
graph?

Answers

Answer:

It means the point x = 5 and y = 10

Answer: The place 5 units right and 10 units up from the center of the graph (the origin).

Step-by-step explanation:

Starting at the origin, the 5 represents moving right 5, and the 10 represents going up 5.

What gravitational force does the moon produce on the Earth if their centers are 3.88x108 m apart and the moon has a mass of 7.34x1022 kg?

Answers

The gravitational force that the moon produces on the Earth is approximately [tex]1.98 \times 10^{20}\ \mathrm{N}$.[/tex]

What is gravitational force?

Gravitational force is the force of attraction that exists between any two objects in the universe with mass. This force is directly proportional to the masses of the objects and inversely proportional to the square of the distance between their centers.

The gravitational force that the moon produces on the Earth can be calculated using the formula:

[tex]F = G \cdot \frac{m_1 \cdot m_2}{r^2}[/tex]

where:

[tex]G$ = gravitational constant = $6.67430 \times 10^{-11}\ \mathrm{N(m/kg)^2}$[/tex]

[tex]m_1$ = mass of the moon = $7.34 \times 10^{22}\ \mathrm{kg}$[/tex]

[tex]m_2$ = mass of the Earth = $5.97 \times 10^{24}\ \mathrm{kg}$ (approximate)[/tex]

[tex]r$ = distance between the centers of the Earth and the moon = $3.88 \times 10^8\ \mathrm{m}$[/tex]

Substituting these values into the formula, we get:

[tex]F &= 6.67430 \times 10^{-11} \cdot \frac{7.34 \times 10^{22} \cdot 5.97 \times 10^{24}}{(3.88 \times 10^8)^2} \&= 1.98 \times 10^{20}\ \mathrm{N}[/tex]

Therefore, the gravitational force that the moon produces on the Earth is approximately [tex]1.98 \times 10^{20}\ \mathrm{N}$.[/tex]

To learn more about gravitational force visit:

https://brainly.com/question/29328661

#SPJ1

Andy has 4 red cards, 3 blue cards, and 2 green cards. He chooses a card and replaces it before choosing a card again. How many possible outcomes are in the sample space of Andy's experiment?
A) 18
B) 9
C)81
D)3

Answers

There are 81 potential outcomes in Andy's sample space.

What are the potential results?

Potential Outcomes is a list of every scenario that could happen as a result of an occurrence. For instance, while rolling a dice, the possible results are 1, 2, 3, 4, 5, and 6. 6. Favorable Result - the intended outcome. For instance, if you roll a 4 on a dice, the only possible result is 4.

The total number of cards (i.e., 4 + 3 + 2 = 9) determines the number of outcomes that can occur in each draw.

We must multiply the total number of results for each draw in order to determine the total number of possible outcomes for the two draws.

For two draws with replacement, there are exactly as many outcomes available as the product of the amount of outcomes that could occur in each draw.

9 × 9 = 81.

To know more about potential outcomes visit:-

https://brainly.com/question/28889662

#SPJ1

The table shows some points on the graph of exponential function g(x)
0 1 2 3 4 g(x) 1 3 9 27 81 What is the range of g?

Answers

the range of g is all positive numbers greater than or equal to 1. In set-builder notation, we can write the range of g as {g(x) | g(x) ≥ 1}.

How to solve and what is graph?

The range of a function refers to the set of all possible output values. Looking at the table, we can see that the output values of g(x) increase rapidly as x increases.

In fact, the output values of g(x) are the result of raising 3 to the power of x, which means that g(x) can never be negative. Therefore, the range of g is all positive numbers greater than or equal to 1. In set-builder notation, we can write the range of g as {g(x) | g(x) ≥ 1}.

A graph is a visual representation of data or mathematical functions. It is a diagram made up of points, lines, and curves that show the relationship between different variables or data points.

Graphs are used to display and analyze data, to illustrate trends and patterns, and to communicate complex information in an easily understandable way. There are many different types of graphs, including bar graphs, line graphs, pie charts, scatter plots, and more, each suited to different types of data and analysis.

To know more about Graph related questions, visit:

https://brainly.com/question/17267403

#SPJ1

what is the answer?
?
No, there is not enough information
Yes, because of the intermediate value theorem

Answers

Because g(x) is continuous on the interval, we can see that the correct option is the last one (counting from the top)

Does the value c exists in the given interval?

Here we have the function g(x), and we know that it is continuous on the interval [1, 6], and that:

g(1) = 18

g(6) = 11

If it is continuous, then g(x) covers all the values between 18 and 11 in the given interval, this means that there must exist a value c in the given interval such that when we evaluat g(x) in that value c, we get the outcome 12, and we know this by the intermediate value theorem.

So the correct optionis the last one.

Learn more about continuous functions at

https://brainly.com/question/18102431

#SPJ1

Work out the size of angle x. 79°) 35​

Answers

Answer: 66

Step-by-step explanation:

all 3 of them should equal to 180

so 79+35 is 114

180-114 will give us the answer which is 66

a
21 units squared
b
27.6 units squared
c
32.2 units squared
d
42 units squared

Answers

The area of the right triangle given in this problem is given as follows:

21 units squared -> Option A.

How to obtain the area of a triangle?

To calculate the area of a triangle, you can use the formula presented as follows:

Area = (1/2) x base x height

In which the parameters are given as follows:

"base" is the length of the side of the triangle that is perpendicular to the height."height" is the length of the perpendicular line segment from the base to the opposite vertex.

For a right triangle, we can consider one side to be the base and the other side to be the height, hence the parameters are given as follows:

Base of 7 units.Height of 6 units.

Hence the area of the triangle is given as follows:

A = 0.5 x 7 x 6 = 21 units squared.

Missing Information

The complete problem is defined as follows:

"Calculate the area of the given triangle".

More can be learned about the area of a triangle at brainly.com/question/21735282

#SPJ1

the circumference of a circle is 4.8m. Calculate the area of the circle​

Answers

Answer:

A≈1.83m²

Step-by-step explanation:

Using the formulas

A=πr2C=2πr

Solving forAA=C24π=4.824·π≈1.83346m²

Answer:

1.83 (approximate)

Step-by-step explanation:

First, we need to find the radius of the circle.

Using the formula:

[tex]C=2 \pi r[/tex]

We have to reorganize terms:

[tex]r=\frac{C}{2\pi}[/tex]

[tex]\frac{4.8}{2 \times \pi}[/tex]

r ≈ 0.76

Now we have the radius and the circumference in order to find the area.

Use the formulas:

[tex]A= \pi r^2[/tex]

[tex]C=2 \pi r[/tex]

A = C^2/4[tex]\pi[/tex]  = 4.8^2 / 4 * pi = 1.83

Question 13 (2 points)
Suppose you flip a coin and then roll a die. You record your result. What is the
probability you flip heads or roll a 3?
1/2
3/4
7/12
1

Answers

Step-by-step explanation:

a probability is always the ratio

desired cases / totally possible cases

we have 2 possible cases for the coin and 6 possible cases for the die.

so, we have 2×6 = 12 combined possible cases :

heads, 1

heads, 2

heads, 3

heads, 4

heads, 5

heads, 6

tails, 1

tails, 2

tails, 3

tails, 4

tails, 5

tails, 6

out of these 12 cases, which ones (how many) are desired ?

all first 6 plus (tails, 3) = 7 cases

so, the correct probability is

7/12

formally that is calculated :

1/2 × 6/6 + 1/2 × 1/6 = 6/12 + 1/12 = 7/12

the probability to get heads combined with the probability to roll anything on the die, plus the probability to get tails combined with the probability to roll 3.

Other Questions
What is the solution for those?And if you can please add the explanations A decline in the number of young people joining the workforce and an increase in the number of active employees who are postponing retirement are examples of _____ forces in the general environment. Converting from decimal to non-decimal bases. info About A number N is given below in decimal format. Compute the representation of N in the indicated base. (a) N = 217, binary. (b) N = 99, hex. (c) N = 344, hex. (d) N =136, base 7. (e) N = 542, base 5. (f) N = 727, base 8. (g) N = 171, hex. (h) N = 91, base 3. (i) N = 840, base 9. When Joselyn went to the store she bought 2.7kg of salt water taffy. What would Joselyn do to find out how many grams she bought?A. Divide by 1000B. Multiply by 1000C. Divide by 100D. Multiply by 100 what can be the advantage of currency undervaluation? group of answer choices increase export increase outward foreign direct investment protect domestic exporting firms protect domestic consumers by giving them more purchasing power over foreign goods and services how has the architecture of the greeks and romans influenced the architecture of modern europe and america (1900-present). include specific examples within your answer. (at least 2 paragraphs or 250 words) what the catholic religious order that founded georgetown university? consider a student loan of $15000 at a fixed APR of 12 % for 20 years TRUE/FALSE. According to research by Stevens and colleagues, when girls are more violent, tolerance for their behavior significantly decreases. translation is accomplished by the interaction of three main components which include mrna, trna, and _____________. what body system moves and supplies nutrients around the body conscious and voluntary movements are associated with the ________ nervous system. When only one lightbulb blows out, an entire string of decorative lights goes out. The lights in this string must be connected ina. parallel with one current pathwayb. parallel with multiple current pathwaysc. series with one current pathwayd. series with multiple current pathways pols 1301 the role of the federal government in the implementation of social policies has decreased since the Each enzyme works best at a specific pH. pH environments below or above this preferred level decrease enzyme activity by causing the enzyme to____ Blood is an example of a basic buffer system. Which of the following could be used to mimic the buffering abilities of blood?Select the correct answer below:HF and NaFCH3NH2 and CH3NH3ClKOH and H2Onone of the above ways in which the medias portrayal of women as vulnerable members ofsociety could lead to greater instances of violence towards women. what is the as surface area of the rectangular prism PLEASE HELP The linear function f(x) = 0.9 + 79 represents the average test score in your math class, where x is the number of the test taken. The linear function g(x) represents the average test score in your science class, where x is the number of the test taken. 1) A white dwarf isA) a precursor to a black hole.B) an early stage of a neutron star.C) what most stars become when they die.D) a brown dwarf that has exhausted its fuel for nuclear fusion.